Question

1) 1) A single-price monopolist is currently producing an output level where P-520, MR = $13, ATC = $15, and MC = $14. In ord
0 0
Add a comment Improve this question Transcribed image text
Answer #1

Answer-B = Profit maximising condition is where MR=MC Here MR<MC, so production should decrease upto when MC=MR - Productio#Please rate positively...thank you

Add a comment
Know the answer?
Add Answer to:
1) 1) A single-price monopolist is currently producing an output level where P-520, MR = $13,...
Your Answer:

Post as a guest

Your Name:

What's your source?

Earn Coins

Coins can be redeemed for fabulous gifts.

Not the answer you're looking for? Ask your own homework help question. Our experts will answer your question WITHIN MINUTES for Free.
Similar Homework Help Questions
  • What happens if a competitive firm is currently producing a level of output at which marginal...

    What happens if a competitive firm is currently producing a level of output at which marginal revenue exceeds marginal cost? Select one: O a. A one-unit decrease in output will increase the firm's profit. b. A one-unit increase in output will increase the firm's profit. O c. Total revenue exceeds total cost. d. Total cost exceeds total revenue.

  • 1. Suppose that a single-price monopolist faces the demand function P 100 Q where I is...

    1. Suppose that a single-price monopolist faces the demand function P 100 Q where I is average weekly household income, and that the firm's marginal cost function is given by MC(Q) 2Q. The firm has no fixed costs. = (a) If the average weekly household income is $600, find the firm's marginal revenue function. (b) What is the firm's profit-maximizing quantity of output? At what price will the firm sell that output? What will the firm's marginal cost be? (c)...

  • Say a monopolist knew that at the current price for its product demand is inelastic. If...

    Say a monopolist knew that at the current price for its product demand is inelastic. If marginal costs for this firm are zero, then in order to maximize profits this monopolist should A increase output. reduce output. keep output at the same level. D decrease its price.

  • The total cost of producing Q units of output to a monopolist is TC(Q) = 2Q2....

    The total cost of producing Q units of output to a monopolist is TC(Q) = 2Q2. The market demand is given byQ= 10−2P. (a) What are the total revenue and marginal revenue functions (b) What price should the monopolist set to maximize its profit? Illustrate your answer on a graph.

  • 1) Which of the following statements regarding a monopolist is correct? A) A monopolist will only...

    1) Which of the following statements regarding a monopolist is correct? A) A monopolist will only produce an output where the demand is perfectly elastic, B) A monopolist will only produce an output where the demand is elastic C) A monopolist will only produce an output where the demand is inelastic. D) A monopolist will only produce an output where the demand is unitary elastic. 2) When is a monopolist's total revenue at a maximum? A) When its marginal revenue...

  • On the basis of the following information for a pure monopolist Product price $500 300 250...

    On the basis of the following information for a pure monopolist Product price $500 300 250 200 150 100 Output Total cost $250 260 290 350 4 5 480 700 27. How many units would the above profit-maximizing monopolist produce a) 1 b) 2 c) 3 d) 4 28. The above monopolist should set its price at: a) $300. b) $250. c) $200 d) $15 29. At its profit-maximizing output, the above monopolist: a) incurs a loss b) Earns an...

  • 1.) The profit-maximizing output level for a monopolist is where the: A. price is maximized. B....

    1.) The profit-maximizing output level for a monopolist is where the: A. price is maximized. B. output sold is maximized. C. ATC curve is minimized. D. maximum efficiency is achieved. E. MR = MC. 2.) An example of price discrimination is the price charged for: A. troll dolls. B. part tickts. C. clothes. D. diamonds.

  • a. A monopolist finds that at its current level of output the marginal cost of production...

    a. A monopolist finds that at its current level of output the marginal cost of production is $9, the average total cost is $9.95,the average variable cost is $9.75 and its marginal revenue is $9.45. What would you recommend that the monopolist do to increase profits? One word answer is not enough. Explain. b.  A monopolist determines that at the current level of output the marginal revenue is $9.00 and its marginal cost is $10. What should the monopolist do to...

  • A monopolist faces the following average revenue (demand) curve:

    Problem 1. (7 points) A monopolist faces the following average revenue (demand) curve: P =  300-0.3Q and the monopolist's cost function is given by C(Q) = 8000+0.3Q2 (a) Derive the monopolist's marginal revenue equation. (2 pts) (b) Derive the monopolist's marginal cost equation. (1 pt) (c) What level of output will the monopolist choose in order to maximize its profits? (2 pts) (d) What price will the monopolist receive at the profit-maximizing level of output? (1 pt) (e) Calculate the monopolist's profit when they produce at the profit-maximizing level....

  • Question 1 (Mandatory) (5 points) Saved If at an output of 10 units a monopolist is...

    Question 1 (Mandatory) (5 points) Saved If at an output of 10 units a monopolist is earning a positive profit, marginal revenue is $6, and marginal cost is $4, then the monopolist A) is in equilibrium. B) should increase output. C) should reduce output. D) should raise the price at the current output level. Question 40 (Mandatory) (5 points) The marginal revenue product of a resource measures OA) the additional cost to a firm of employing one more unit of...

ADVERTISEMENT
Free Homework Help App
Download From Google Play
Scan Your Homework
to Get Instant Free Answers
Need Online Homework Help?
Ask a Question
Get Answers For Free
Most questions answered within 3 hours.
ADVERTISEMENT
ADVERTISEMENT
ADVERTISEMENT